LSAT and Law School Admissions Forum

Get expert LSAT preparation and law school admissions advice from PowerScore Test Preparation.

 rachue
  • Posts: 140
  • Joined: Jun 22, 2011
|
#1534
Hi there,

I'm having a hard time understanding the differences between A, B, and E. How should this be diagrammed to come to the correct answer? It is very confusing and at face value each of these answers appear to be saying the same thing!

Thanks in advance.
 Nikki Siclunov
PowerScore Staff
  • PowerScore Staff
  • Posts: 1362
  • Joined: Aug 02, 2011
|
#1540
You should be looking for an answer choice that states the following:

If Leaders of all major parties DO NOT support the bill ==> Bill will not pass

Answer choice (A) states that most bills that have not been supported by EVEN ONE leader of a major party have not been passed into law. This answer choice strengthens the conclusion, because the sufficient condition in that statement matches the situation described in the stimulus: the leaders of ALL major parties oppose the bill. Also, the prediction in the stimulus is consistent with the necessary condition: the bill does not pass.

Note that this answer does not fully justify the conclusion, since it provides information about the past (whereas the conclusion of the argument seeks to predict the future). Nevertheless, it does strengthen the conclusion, and is therefore correct.

Answer choice (B) is a Mistaken Reversal of the answer you are looking for:

Bill not pass ==> Leaders of all major parties oppose it

Answer choice (E) is attractive, but incorrect. Note that the contrapositive of the statement you are looking for is this:

Bill DOES pass ==> Leaders of ALL major parties DO support the bill

However, answer choice (E) states that most bills that have been passed into law were supported by AT LEAST one leader of a major party. Although this statement may be consistent with the contrapositive of the statement we are looking for, it does not add any support to the conclusion of the argument.
 rachue
  • Posts: 140
  • Joined: Jun 22, 2011
|
#1544
Ah, OK. So if I run into a problem like this again, my first step should be to prephrase the answer into a conditional statement if possible, and then look for the closest answer? What really threw me off with this questions was that the wording was so similar in each of the answers.
 Nikki Siclunov
PowerScore Staff
  • PowerScore Staff
  • Posts: 1362
  • Joined: Aug 02, 2011
|
#1558
Yes, absolutely. Otherwise, you will find many of the answers to be equally attractive and indistinguishable from one another. Your prephrase will give you a lens through which to examine them more closely.
 rachue
  • Posts: 140
  • Joined: Jun 22, 2011
|
#1572
Great, thanks!
 lawschoolforme
  • Posts: 33
  • Joined: Oct 15, 2013
|
#13089
Hello,

I'm having trouble making sense of this question and am having trouble understanding how to approach it.

I know this is a strengthen question and that, with this particular stem, I'm looking for an answer choice that creates a situation that reinforces the sense that "the new agriculture bill will almost surely fail to pass". But the answer choices are super confusing to me! Should I be logically mapping this out?

Sincerely,
lawschoolforme
 Lucas Moreau
PowerScore Staff
  • PowerScore Staff
  • Posts: 216
  • Joined: Dec 13, 2012
|
#13096
Hello, LSFM,

The trick with Strengthen questions is that you're not just trying to support the conclusion - you're trying to support the link between the premise and the conclusion. It's the exact opposite of a Weaken question - there, you're not just trying to shoot down the conclusion, you are trying to show that the premises do not lead to the conclusion.

In this case, the conclusion is "The bill won't pass" and the premise is "No major party leaders support it." A is the best answer choice because it reinforces the specific fact that the lack of support by major party leaders is what will cause the bill to not pass, not just because it bolsters the idea that the bill will not pass by itself.

Hope that helps,
Lucas Moreau
PowerScore
 lawschoolforme
  • Posts: 33
  • Joined: Oct 15, 2013
|
#13313
Lucas,

Thanks again so much for your explanation! I see better now what kind of answer choice I should be looking for.

At the same time, I'm still a little puzzled at how to tell the difference between A and B. Er...I know they must be saying different things...but I'm having trouble figuring out how to write their logical equivalents.

Thanks again for your help!

Sincerely,
lawschoolforme
 Ron Gore
PowerScore Staff
  • PowerScore Staff
  • Posts: 220
  • Joined: May 15, 2013
|
#13322
Hey there, LSFM!

Answer choices (A) and (B) are definitely tricky. My bottom line answer is that they are actually talking about two different sets of bills.

Answer choice (A) says: "most bills that have not been supported by even one leader of a major party have not been passed into law." The subject of this sentence is "bills that have not been supported by even one leader."

Answer choice (B) says: "most bills that have not been passed into law were not supported by even one member of a major party." The subject of this sentence is "bills that have not been passed into law."

So, these choices refer to two different sets of bills. The evidence given in the stimulus is that the leaders of all the major parties oppose the bill. Answer choice (A) gives the historical data, basically an unspoken rule the author is applying, to reach the conclusion that the bill will not pass. So, adding the rule from (A) into the argument:

Premise: The leaders of all the major parties oppose the bill.

Rule: Most bills that have not been supported by even one leader of a major party have passed into law (i.e., the leaders of all the major parties oppose the bill)

Conclusion: Thus, the new agriculture bill will almost surely fail to pass.

This still is a weak argument, because the author is committing a time shift error to use the historical data as a rule to reach a predictive conclusion about the passage of this particular bill. But, answer choice (A) is correct because it provides the rule implicitly, if improperly, applied by the author to arrive at the conclusion based on the evidence given in the premise.

Contrast this with answer choice (B), which does not connect as well with the evidence. It says that if you look at all the bills that didn't pass into law, most of them did not have the support of even one leader of a major party. Answer choice (A) is just looking at the bills that didn't have the support of even one leader, instead of looking at all the bills that didn't pass into law.

Please let me know if I can help you further.

Thanks!

Ron :-D
 lawschoolforme
  • Posts: 33
  • Joined: Oct 15, 2013
|
#13342
Hey Ron,

Oh damnsals - that makes so much sense! THANK YOU!

-lawschoolforme

Get the most out of your LSAT Prep Plus subscription.

Analyze and track your performance with our Testing and Analytics Package.